Quiz on Math Set 140

Score: 00

Home / Quiz / Quiz on Math / Set 140

Welcome, Guest ! This quiz is on Math. Each question has four options. Only one option is correct. You can select any option. If your selected option is correct then the option will be lightned with green color. If your option is incorrect then the option will be lightned with red color.

One correct question = 100 Marks. You will be able to see your marks on the top.

You are attempting this quiz as Guest. Data will not saved in database. To save data for future reference, attempt quizzes after login.

Note: You can select only one option out of four option of each question.

Share Quiz on WhatsApp

Quiz on Math Set 140 :

Question 1 :

The LCM of two numbers is 44 times of their of HCF. The sum of the LCM and HCF is 1125. If one number is 25 then the other number is = ?
A. 1100
B. 975
C. 900
D. 800

Correct Ans: Option A

Question 2 :

The LCM of three different numbers is 120. Which of the following cannot be their HCF ?
A. 8
B. 12
C. 24
D. 35

Correct Ans: Option D

Question 3 :

The greatest number for four digits which when divided by 12, 16 and 24 leave remainder 2,6 and 14 respectively is = ?
A. 9974
B. 9970
C. 9807
D. 9998

Correct Ans: Option A

Question 4 :

Two numbers are in the ratio 3 : 4. The product of their HCF and LCM is 2028. The sum of the numbers is = ?
A. 68
B. 72
C. 86
D. 91

Correct Ans: Option D

Question 5 :

Sum of two numbers is 384. HCF of the numbers is 48. The difference of the numbers is = ?
A. 100
B. 192
C. 288
D. 336

Correct Ans: Option C

Question 6 :

The HCF and LCM of two numbers are 21 and 4641 respectively. If one of the numbers lies between 200 and 300 , the two numbers are ?
A. 273, 357
B. 273, 359
C. 273, 361
D. 273, 363

Correct Ans: Option A

Question 7 :

The LCM of two multiples of 12 is 1056. If one of the number is 132, the other number is ?
A. 12
B. 72
C. 96
D. 132

Correct Ans: Option C

Question 8 :

How many rotations will the hour hand of a clock complete in 72 hours?
A. 3
B. 6
C. 9
D. 12

Correct Ans: Option B

Question 9 :

At what time, in minutes, between 3 O' clock and 4 O' clock, both the needles will coincide each other?
A. 5 1/11" past
B. 12 4/11" past
C. 13 4/11" past
D. 16 4/11" past

Correct Ans: Option D

Question 10 :

The hands of a clock are 10 cm and 7 cm respectively. The difference between the distance traversed by their extremities in 3 days 5 hours is
A. 4552.67 cm
B. 4555.67 cm
C. 4557.67 cm
D. 4559.67 cm

Correct Ans: Option C

Quiz Summary


Here the correct answers of every question in this quiz.

Q.1 [A],     Q.2 [D],     Q.3 [A],     Q.4 [D],     Q.5 [C],     Q.6 [A],     Q.7 [C],     Q.8 [B],     Q.9 [D],     Q.10 [C],    


Total Questions: 10
Your score:
Subject: Math
Share Quiz on WhatsApp